The Coq FAQ says that function extensionality is consistent with predicative Set
. It's not fully clear to me from this whether it's consistent with impredicative Set
(or maybe the consistency is unknown in that case).
Asked
Active
Viewed 372 times
30

András Kovács
- 29,931
- 3
- 53
- 99
-
2This sounds a bit misleading; I imagine that it is consistent with impredicative Set as well. But that's probably a better question for the Coq mailing list. – Arthur Azevedo De Amorim Nov 03 '16 at 07:49
-
1Additionally, this question is about the theory of Coq rather than a programming issue with Coq and should probably be migrated to MathOverflow where it has more chances of being answered in details. – Zimm i48 Nov 03 '16 at 10:53
-
2If you're considering migration then [cstheory.SE](http://cstheory.stackexchange.com) would be another good choice, imo. – Anton Trunov Nov 03 '16 at 15:47
-
This question has been mentioned by [Zimm i48](http://stackoverflow.com/users/3335288/zimm-i48) in [this](https://sympa.inria.fr/sympa/arc/coq-club/2016-12/msg00114.html) related Coq-Club thread ([this](https://sympa.inria.fr/sympa/arc/coq-club/2016-12/msg00115.html) post specifically). – Anton Trunov Dec 21 '16 at 14:39